2022 AMC 10B Problems/Problem 14: Difference between revisions
Mathisfun21 (talk | contribs) |
Pinotation (talk | contribs) |
||
| (40 intermediate revisions by 19 users not shown) | |||
| Line 1: | Line 1: | ||
==Problem== | ==Problem== | ||
Suppose that < | Suppose that <imath>S</imath> is a subset of <imath>\left\{ 1, 2, 3, \ldots , 25 \right\}</imath> such that the sum of any two (not necessarily distinct) elements of <imath>S</imath> is never an element of <imath>S.</imath> What is the maximum number of elements <imath>S</imath> may contain? | ||
elements of < | |||
<imath>\textbf{(A)}\ 12 \qquad\textbf{(B)}\ 13 \qquad\textbf{(C)}\ 14 \qquad\textbf{(D)}\ 15 \qquad\textbf{(E)}\ 16</imath> | |||
==Solution 1 (Pigeonhole Principle)== | |||
We categorize numbers < | Let <imath>M</imath> be the largest number in <imath>S</imath>. | ||
We categorize numbers <imath>\left\{ 1, 2, \ldots , M-1 \right\}</imath> (except <imath>\frac{M}{2}</imath> if <imath>M</imath> is even) into <imath>\left\lfloor \frac{M-1}{2} \right\rfloor</imath> groups, such that the <imath>i</imath>th group contains two numbers <imath>i</imath> and <imath>M-i</imath>. | |||
Recall that < | Recall that <imath>M \in S</imath> and the sum of two numbers in <imath>S</imath> cannot be equal to <imath>M</imath>, and the sum of numbers in each group above is equal to <imath>S</imath>. Thus, each of the above <imath>\left\lfloor \frac{M-1}{2} \right\rfloor</imath> groups can have at most one number in <imath>S</imath>. | ||
Therefore, | Therefore, | ||
<cmath> | <cmath> | ||
| Line 15: | Line 15: | ||
|S| & \leq 1 + \left\lfloor \frac{M-1}{2} \right\rfloor \\ | |S| & \leq 1 + \left\lfloor \frac{M-1}{2} \right\rfloor \\ | ||
& \leq 1 + \left\lfloor \frac{25}{2} \right\rfloor \\ | & \leq 1 + \left\lfloor \frac{25}{2} \right\rfloor \\ | ||
& = 13 . | & = 13. | ||
\end{align*} | \end{align*} | ||
</cmath> | </cmath> | ||
Next, we construct an instance of < | Next, we construct an instance of <imath>S</imath> with <imath>|S| = 13</imath>. | ||
Let < | Let <imath>S = \left\{ 13, 14, \ldots , 25 \right\}</imath>. | ||
Thus, this set is feasible. | Thus, this set is feasible. | ||
Therefore, the most number of elements in < | Therefore, the most number of elements in <imath>S</imath> is | ||
< | <imath>\boxed{\textbf{(B) }13}</imath>. | ||
~Steven Chen (Professor Chen Education Palace, www.professorchenedu.com) | ~Steven Chen (Professor Chen Education Palace, www.professorchenedu.com) | ||
== Solution 2 (Pigeonhole v2) == | |||
We construct a possible subset <imath>S</imath> with <imath>13</imath> elements by including all odd integers from <imath>1</imath> to <imath>25</imath>, inclusive. <imath>S=\left\{ 1, 3, 5, \cdots , 25 \right\}</imath>. The sum of any <imath>2</imath> elements is even, and thus cannot be an element of <imath>S</imath>. | |||
To show that <imath>S</imath> cannot have more than <imath>13</imath> elements, assume for sake of contradiction that <imath>|S| \geq 14</imath>. Let <imath>S=\left\{ x_1, x_2, \cdots , x_n \right\}</imath> where <imath>n \geq 14</imath> and <imath>x_1 < x_2 < \cdots < x_n</imath>. Because the sums of any <imath>2</imath> (not necessarily distinct) elements do not appear in <imath>S</imath>, <imath>x_1+x_i</imath> is not an element of <imath>S</imath> for all <imath>1 \leq i \leq n</imath>. So, <imath>x_1, x_2, \cdots , x_n , x_1+x_1, x_1+x_2, \cdots , x_1+x_n</imath> are all distinct integers. Let these integers be elements of the set <imath>T</imath>. <imath>|T|=2n</imath>, and because <imath>n \geq 14</imath>, <imath>|T| \geq 28</imath>. But all elements of <imath>T</imath> must be <imath>\geq x_1</imath> and <imath>\leq x_1+x_n \leq x_1+25</imath>, leaving only 26 possible values for the elements in <imath>T</imath>. By the Pigeonhole Principle, the elements cannot be distinct, and we have a contradiction. | |||
Thus, <imath> \boxed{\textbf{(B) }13}</imath> is the maximum possible size of <imath>S</imath>. | |||
~starwars101 | |||
==Solution 3 (Intuitive and Fast)== | |||
Notice how \( 24 + 25 > 25 \), \( 23 + 24 > 25 \), and this continues backward until \( n + n+1 > 25 \). We then see that \( 2n+1 > 25 \rightarrow 2n > 24 \rightarrow n > 12 \). | |||
Thus the elements in the set must have the property \( n > 12 : n \in \mathbb{Z}^+ \) and therefore our set is \( \{13, 14, 15, \dots, 25\} \) in which there is \( 25 - 13 + 1 \Rightarrow \) <imath> \boxed{\textbf{(B) }13}</imath> elements. | |||
~Pinotation | |||
==Video Solution== | ==Video Solution== | ||
| Line 32: | Line 50: | ||
~Steven Chen (Professor Chen Education Palace, www.professorchenedu.com) | ~Steven Chen (Professor Chen Education Palace, www.professorchenedu.com) | ||
==Video Solution by Interstigation== | |||
https://youtu.be/in3N3Os5kGw | |||
==Video Solution by TheBeautyofMath== | |||
https://youtu.be/Mi2AxPhnRno?t=1056 | |||
~IceMatrix | |||
== See Also == | |||
[https://artofproblemsolving.com/wiki/index.php?title=2025_AMC_12A_Problems/Problem_15 2025 AMC 10A Q21/AMC 12A Q15] | |||
{{AMC10 box|year=2022|ab=B|num-b=13|num-a=15}} | |||
{{MAA Notice}} | |||
Latest revision as of 15:56, 9 November 2025
Problem
Suppose that
is a subset of
such that the sum of any two (not necessarily distinct) elements of
is never an element of
What is the maximum number of elements
may contain?
Solution 1 (Pigeonhole Principle)
Let
be the largest number in
.
We categorize numbers
(except
if
is even) into
groups, such that the
th group contains two numbers
and
.
Recall that
and the sum of two numbers in
cannot be equal to
, and the sum of numbers in each group above is equal to
. Thus, each of the above
groups can have at most one number in
.
Therefore,
Next, we construct an instance of
with
.
Let
.
Thus, this set is feasible.
Therefore, the most number of elements in
is
.
~Steven Chen (Professor Chen Education Palace, www.professorchenedu.com)
Solution 2 (Pigeonhole v2)
We construct a possible subset
with
elements by including all odd integers from
to
, inclusive.
. The sum of any
elements is even, and thus cannot be an element of
.
To show that
cannot have more than
elements, assume for sake of contradiction that
. Let
where
and
. Because the sums of any
(not necessarily distinct) elements do not appear in
,
is not an element of
for all
. So,
are all distinct integers. Let these integers be elements of the set
.
, and because
,
. But all elements of
must be
and
, leaving only 26 possible values for the elements in
. By the Pigeonhole Principle, the elements cannot be distinct, and we have a contradiction.
Thus,
is the maximum possible size of
.
~starwars101
Solution 3 (Intuitive and Fast)
Notice how \( 24 + 25 > 25 \), \( 23 + 24 > 25 \), and this continues backward until \( n + n+1 > 25 \). We then see that \( 2n+1 > 25 \rightarrow 2n > 24 \rightarrow n > 12 \).
Thus the elements in the set must have the property \( n > 12 : n \in \mathbb{Z}^+ \) and therefore our set is \( \{13, 14, 15, \dots, 25\} \) in which there is \( 25 - 13 + 1 \Rightarrow \)
elements.
~Pinotation
Video Solution
~Steven Chen (Professor Chen Education Palace, www.professorchenedu.com)
Video Solution by Interstigation
Video Solution by TheBeautyofMath
https://youtu.be/Mi2AxPhnRno?t=1056
~IceMatrix
See Also
| 2022 AMC 10B (Problems • Answer Key • Resources) | ||
| Preceded by Problem 13 |
Followed by Problem 15 | |
| 1 • 2 • 3 • 4 • 5 • 6 • 7 • 8 • 9 • 10 • 11 • 12 • 13 • 14 • 15 • 16 • 17 • 18 • 19 • 20 • 21 • 22 • 23 • 24 • 25 | ||
| All AMC 10 Problems and Solutions | ||
These problems are copyrighted © by the Mathematical Association of America.